LSAT and Law School Admissions Forum

Get expert LSAT preparation and law school admissions advice from PowerScore Test Preparation.

 Administrator
PowerScore Staff
  • PowerScore Staff
  • Posts: 8916
  • Joined: Feb 02, 2011
|
#32463
Complete Question Explanation

Parallel Flaw. The correct answer choice is (C)

The author of this stimulus begins with the “some people say...” technique, offering the view of “some philosophers” that when we visually perceive an object, we form a mental image of that object in our minds. Predictably, the author disagrees with this view.

According to the author, this view of visual perception requires an infinite regress—which can be defined as “a sequence of reasoning or justification that can never come to an end”—of mental images. Because an infinite regress is absurd, the author tells us, the philosophers’ hypothesis cannot be correct.

The question stem indicates that this is a Parallel Reasoning question. Our prephrase is that the author introduces another’s theory, states as a premise that the theory leads to an absurd result and, on that basis, concludes definitively that the theory cannot be correct. Given the premise that the theory leads to an absurd result, the conclusion that the theory must be incorrect is valid. The correct answer choice will contain an argument with this same structure, though in a different context.

Answer choice (A): While this argument introduces a theory and reaches a definitive conclusion, it is incorrect for two reasons. First, the conclusion is invalid, because it distorts the linguists’ view. The linguists claimed that “many of the world’s languages can be traced back to a common source,” Indo-European. However, the conclusion is that “Indo-European cannot be the earliest language.” This conclusion distorts the linguists’ view, because the linguists did not claim that Indo-European is the earliest language. Next, the premises do not match the stimulus. In the stimulus, the premises point out that the theory requires an absurd result. In this answer, the premises indicate that the theory is inconsistent with the evidence.

Answer choice (B): This is a Shell Game answer choice, in which a term from the stimulus is used in a different context to confuse you. The stimulus referred to the “infinite regress” required by the theory. Here, a premise states that there are an infinite number of theories, each of which account for the data. Although the conclusion is similar to that in the stimulus, that a stated theory cannot be correct, the premises do not match, meaning this choice fails the Match the Premises test.

Answer choice (C): This is the correct answer choice, because its logical structure is most similar to that of the argument in the stimulus. A theory is presented, and we are told that this theory requires an infinite regress. Although the term “infinite regress” is not used, were told about a cycle of precedent theories, “and so on, without end.” Because this result is impossible, the argument concludes that the theory cannot be correct.

Answer choice (D): The conclusion in this argument, that the “definition is unfortunate,” does not match the conclusion in the stimulus, that the theory “cannot be correct” or, as in answer choice (C), the theory “must be false.” so, this answer fails the Match the Conclusion test.

Answer choice (E): Even though this argument introduces a theory and then concludes the theory is incorrect, the support for that theory does not match the premises in the stimulus, which established that the theory required an infinite regress. Here, the support for the conclusion is that existing evidence contradicts the theory. This answer choice fails the Match the Premises test.

Get the most out of your LSAT Prep Plus subscription.

Analyze and track your performance with our Testing and Analytics Package.